ChaseDream
搜索
12
返回列表 发新帖
楼主: vivian8389
打印 上一主题 下一主题

求救。这道逻辑怎么都看不懂。GWD6-Q38

[复制链接]
11#
发表于 2011-6-13 14:15:55 | 只看该作者
Main conclusion: If the governmnet regulates the industry, only the three large companies will be able to remain in business.

A) None of the three large companies will go out of business if the government does NOT regulate the manufacture of the product.

If you negate A, you have:
SOME of the three large companies will go out of business if the government does NOT regulate the manufacture of the product.
If this is true, what would happen to the main conclusion of the argument? Nothing. Because after negation, the trigger of negated A is: if the government does NOT regulate the manufacture of the product. And the trigger for the main conclusion is: If the governmnet regulates the industry. No trigger, no effect in formal logic. A is not necessary.
-- by 会员 sdcar2010 (2011/6/13 11:43:53)


啊!!对!我把条件给改了!
那如果A是“None of the three large companies will go out of business if the government regulate the manufacture of the product.”那么和E比较起来,我应该选那个呢~~嘿嘿
12#
发表于 2011-6-13 19:39:45 | 只看该作者
Main conclusion: If the governmnet regulates the industry, only the three large companies will be able to remain in business.

A) None of the three large companies will go out of business if the government does NOT regulate the manufacture of the product.

If you negate A, you have:
SOME of the three large companies will go out of business if the government does NOT regulate the manufacture of the product.
If this is true, what would happen to the main conclusion of the argument? Nothing. Because after negation, the trigger of negated A is: if the government does NOT regulate the manufacture of the product. And the trigger for the main conclusion is: If the governmnet regulates the industry. No trigger, no effect in formal logic. A is not necessary.
-- by 会员 sdcar2010 (2011/6/13 11:43:53)



啊!!对!我把条件给改了!
那如果A是“None of the three large companies will go out of business if the government regulate the manufacture of the product.”那么和E比较起来,我应该选那个呢~~嘿嘿
-- by 会员 fangsigaoa (2011/6/13 14:15:55)


That's tough. Both have merits.
13#
发表于 2011-7-7 21:37:11 | 只看该作者
Main conclusion: If the governmnet regulates the industry, only the three large companies will be able to remain in business.

A) None of the three large companies will go out of business if the government does NOT regulate the manufacture of the product.

If you negate A, you have:
SOME of the three large companies will go out of business if the government does NOT regulate the manufacture of the product.
If this is true, what would happen to the main conclusion of the argument? Nothing. Because after negation, the trigger of negated A is: if the government does NOT regulate the manufacture of the product. And the trigger for the main conclusion is: If the governmnet regulates the industry. No trigger, no effect in formal logic. A is not necessary.
-- by 会员 sdcar2010 (2011/6/13 11:43:53)



Can I consider it out of scope? Because the argument concerns the condition that government regulates the manufacture.
However, can i transfer the double denial sentence into an affirmative sentence, that is, SOME of the three large companies will not go out of business if the government does NOT regulate the manufacture of the product. This condition seems to weaken the argument.
14#
发表于 2011-7-7 21:57:13 | 只看该作者
E is right.

What the small companies could not afford is the cost of converting their production lines to a new set of manufacturing specifications.  It has nothing to do with what their current manufacturing specifications are. Before the announcement of the final manufacturing specifications, no one knows for sure if the current specifications used by the small companies meet the standard.

B is irrelevant to the argument since we do NOT know if small companies NEED to convert.

From the stimulus, the author PRESUMED that the small companies have to convert their production line in order to meet the new manufacturing specification.  That's why the author concluded that only the big companies can survive.  What if the small companies have already met the new manufacture specification with their current production line?  This will obliviate the need for such costly conversion for the small companies.
-- by 会员 sdcar2010 (2011/5/29 22:02:06)



how about the explanation concern B?
(Richie: Because the argument explicitly argues that one of the seven companies can afford to convert their production. This choice serves the same function as the cause in the argument. Even if small companies cost more, it is nothing to do with the conclusion.)
15#
发表于 2011-8-19 15:43:10 | 只看该作者
再问一下,B为什么不对呢?小公司regulate的花费比大公司多的话不也能削弱麽?
-- by 会员 木易爱青 (2011/5/29 19:14:37)


如果都需要修改,修改的花费是一样的,只是大公司财力丰厚,can afford,小公司不行而已。
16#
发表于 2011-9-17 16:24:50 | 只看该作者
GWD6-Q38:
Three large companies and seven small companies currently manufacture a product with potential military applications.If the government regulates the industry, it will institute a single set of manufacturing specifications to which all ten companies will have to adhere.In this case, therefore, since none of the seven small companies can afford to convert their production lines to a new set of manufacturing specifications, only the three large companies will be able to remain in business.

Which of the following is an assumption on which the author’s argument relies?
A.None of the three large companies will go out of business if the government does not regulate the manufacture of the product.
B.It would cost more to convert the production lines of the small companies to a new set of manufacturing specifications than it would to convert the production lines of the large companies.
C.Industry lobbyists will be unable to dissuade the government from regulating the industry.
D.Assembly of the product produced according to government manufacturing specifications would be more complex than current assembly procedures.
E.None of the seven small companies currently manufactures the product to a set of specifications that would match those the government would institute if the industry were to be regulated.
这道题没思路,主要是逻辑链不清楚,很纠结,请大牛指点
-- by 会员 vivian8389 (2011/4/16 15:18:32)



我也没大看懂,只是觉得可以直接来做~你看,题目中therefore后面说,因为none of 这七个小公司balabala,所以只有那三个大公司balabala~问题问一个assumption      那么这个assumption肯定是与那七 个小公司有关的,所以选E
您需要登录后才可以回帖 登录 | 立即注册

Mark一下! 看一下! 顶楼主! 感谢分享! 快速回复:

手机版|ChaseDream|GMT+8, 2024-9-19 10:19
京公网安备11010202008513号 京ICP证101109号 京ICP备12012021号

ChaseDream 论坛

© 2003-2023 ChaseDream.com. All Rights Reserved.

返回顶部